K
Khách

Hãy nhập câu hỏi của bạn vào đây, nếu là tài khoản VIP, bạn sẽ được ưu tiên trả lời.

NM
14 tháng 8 2021

A H B C M

ta có : \(sinB=\frac{AH}{AB}\Rightarrow AH=24\times\frac{5}{13}=\frac{120}{13}cm\)

\(sinB=\frac{5}{13}\Rightarrow tanB=\frac{5}{12}\)

mà \(tanB=\frac{AC}{AB}\Rightarrow AC=AB.tanB=24\times\frac{5}{12}=10cn\)

\(\Rightarrow AM=5cm\Rightarrow BM=\sqrt{AM^2+AB^2}=\sqrt{25+24^2}=\sqrt{601}cm\)

19 tháng 10 2023

loading...   

\(tanB=\dfrac{AC}{AB}=\dfrac{5}{12}\)

⇒ AC = \(\dfrac{5}{12}\) .AB

= \(\dfrac{5}{12}.5\)

\(=\dfrac{25}{12}\) (cm)

∆ABC vuông tại A

⇒ BC² = AB² + AC² (Pytago)

\(=5^2+\left(\dfrac{25}{12}\right)^2\)

= \(\dfrac{4225}{144}\)

⇒ BC = \(\dfrac{65}{12}\) (cm)

AH.BC = AB.AC

⇒ AH = AB . AC : BC

= 5 . \(\dfrac{25}{12}:\dfrac{65}{12}\)

\(=\dfrac{25}{13}\left(cm\right)\)

M là trung điểm của AC

⇒ AM = AC : 2 = \(\dfrac{25}{12}:2\) \(=\dfrac{25}{24}\) (cm)

∆ABM vuông tại A

⇒ BM² = AB² + AM²

= \(5^2+\left(\dfrac{25}{24}\right)^2\)

= \(\dfrac{15025}{576}\)

⇒ BM = \(\dfrac{5\sqrt{601}}{24}\) (cm)

10 tháng 5 2018

Áp dụng tỉ số tanB trong tam giác vuông HAB và các hệ thức lượng trong tam giác vuông, chúng ta tính được AC = 30 13 cm; BM = 601 4 cm

a: XétΔABH vuông tại H và ΔACH vuông tại H có

AB=AC
AH chung

Do đó: ΔABH=ΔACH

Suy ra: BH=CH

b: BH=CH=BC/2=18(cm)

nên AH=24(cm)

NV
25 tháng 7 2021

Áp dụng định lý Pitago:

\(BC=\sqrt{AB^2+AC^2}=25\left(cm\right)\)

Áp dụng hệ thức lượng:

\(AH.BC=AB.AC\Rightarrow AH=\dfrac{AB.AC}{BC}=12\left(cm\right)\)

Do AM là trung tuyến ứng với cạnh huyền

\(\Rightarrow AM=\dfrac{1}{2}BC=\dfrac{25}{2}=12,5\left(cm\right)\)

Áp dụng định lí Pytago tam giác ABC vuông tại A

\(BC^2=225+400=625\Rightarrow BC=25\)cm 

Xét tam giác ABC, đường cao AH 

* Áp dụng hệ thức : \(AH.BC=AB.AC\Rightarrow AH=\dfrac{AB.AC}{BC}=\dfrac{300}{25}=12\)cm 

Vì AM là đường trung tuyến suy ra : \(AM=\dfrac{BC}{2}=\dfrac{25}{2}\)cm 

26 tháng 2 2017

a ,   Δ A B C ,   A ⏜ = 90 0 , A H ⊥ B C g t ⇒ A H = B H . C H = 4.9 = 6 c m Δ A B H ,   H ⏜ = 90 0   g t ⇒ tan B = A H B H = 6 4 ⇒ B ⏜ ≈ 56 , 3 0 b ,   Δ A B C ,   A ⏜ = 90 0 , M B = M C g t ⇒ A M = 1 2 B C = 1 2 .13 = 6 , 5 c m S Δ A H M = 1 2 M H . A H = 1 2 .2 , 5.6 = 7 , 5 c m 2

a: Ta có: \(\sin\widehat{B}=\dfrac{1}{3}\)

nên \(\dfrac{AC}{BC}=\dfrac{1}{3}\)

hay BC=3AC

Áp dụng định lí Pytago vào ΔABC vuông tại A, ta được:

\(BC^2=AB^2+AC^2\)

\(\Leftrightarrow8\cdot AC^2=16\)

\(\Leftrightarrow AC=\sqrt{2}cm\)

\(\Leftrightarrow BC=3\sqrt{2}cm\)

\(\Leftrightarrow AH=\dfrac{4\sqrt{2}}{3\sqrt{2}}=\dfrac{4}{3}cm\)

b: \(\cos\widehat{MAH}=\dfrac{AH}{AM}=\dfrac{4}{3}:\dfrac{3\sqrt{2}}{2}=\dfrac{4}{3}\cdot\dfrac{2}{3\sqrt{2}}=\dfrac{8\sqrt{2}}{18}=\dfrac{4\sqrt{2}}{9}\)

7 tháng 9 2017

Chọn D.

Gọi M là trung điểm của AC suy ra

 .

Do tam giác BAM vuông tại A